Folgt Mikrokausalität aus der Lorentz-Invarianz?

Gilt Mikrokausalität in einer Lorentz-Invarianten-Theorie automatisch? In einer freien Theorie ist dies offensichtlich. In einer interagierenden Theorie fand ich einige 'Beweise' in diesem Artikel: http://arxiv.org/abs/0709.1483

Allerdings zeigen die Beweise, dass für raumartig getrennt X Und j

0 | [ ϕ ^ ( X ) , ϕ ^ ( j ) ] | 0 = 0.

Aber damit die Bedingung der Mikrokausalität auf Operatorebene gilt, müssen wir das zeigen

N | [ ϕ ^ ( X ) , ϕ ^ ( j ) ] | N = 0 N

Wo | N bildet eine Grundlage. Meine Frage ist, kann man das zeigen? Oder sind weitere Annahmen erforderlich?

Antworten (2)

Wenn alle abgeschnitten N -Punktfunktionen verschwinden für N > 2 (dh wir haben es mit einem sogenannten verallgemeinerten Freifeld zu tun ), Mikrokausalität für Vakuumerwartungswerte und auf Betreiberebene sind äquivalent. Ersteres wiederum folgt allein aus der Lorentz-Invarianz im Fall von skalaren (aber nicht notwendigerweise freien) Körpern, wie Pierre-Denis Methée, ein Schüler von de Rham (Sur les Distributions Invarianten dans le Groupe de Rotations) . de Lorentz , Commentarii Mathematici Helvetici 28 (1954) 225-269). Wenn das Feld wechselwirkt, ist dies nicht mehr der Fall, und tatsächlich folgt die Mikrokausalität nicht allein aus der Lorentz-Invarianz, auch wenn die positive Eindeutigkeit der N -Punktfunktionen und die Energie-Impuls-Spektrum-Bedingung gelten ebenfalls. Bearbeiten (15. Juni 2022): Wie Nanashi No Gombe in den Kommentaren unten betonte, liefern parastatistische Feldmodelle ein Beispiel dafür , die sich nicht auf den Bose/Fermi-Begriff der Mikrokausalität festlegen müssen, aber dennoch Lorentz-kovariant sein können.

Es ist auch wichtig darauf hinzuweisen, dass es nicht-Lorentz-invariante Quantenfeldtheorien gibt, die dennoch mikrokausal sind (z. B. einige Modelle, die an ein geeignetes externes „Äther“-Feld gekoppelt sind). In solchen Modellen reichen die Mikrokausalität und die Energie-Impuls-Spektrum-Bedingung aus, um sicherzustellen, dass das Energie-Impuls-Spektrum eine Lorentz-invariante Form hat und daher Lorentz-invariante Dispersionsgesetze hat (dh entweder vom Typ "Masse-Hülle" oder "Lichtkegel") ), auch ohne echte Lorentz-Invarianz - dies ist eine Folge der Jost-Lehmann-Dyson-Darstellung der Zweipunktfunktion, die nicht auf Lorentz-Invarianz beruht. Einmal mehr. dies zeigt, dass die Konzepte der Lorentz-Invarianz und der Mikrokausalität in einer Quantenfeldtheorie nicht äquivalent sind.

Danke für deine Antwort. Weinberg sagt tatsächlich, dass wir Mikrokausalität brauchen, um die Lorentz-Invarianz der S-Matrix zu beweisen. Er sagt, dass dies für etwas wie das Dirac-Feld, das nicht direkt beobachtbar ist, die einzige „Rechtfertigung“ der Mikrokausalitätsbedingung ist. Das von mir verlinkte Papier behauptet jedoch, dass Mikrokausalität aus der Lorentz-Invarianz folgen kann. Sie zeigen jedoch, dass es auch in gekrümmter Raumzeit gilt, in Abwesenheit von globalem LI. Es ist tatsächlich ein ziemlich interessantes Papier.
Es ist wichtig anzumerken, dass die Argumente in dem von Ihnen zitierten Artikel nicht streng sind und hauptsächlich auf formalen funktionalen Integralen und Störungstheorie basieren. Das von mir zitierte Ergebnis von Methée ist dagegen völlig streng und gilt unter sehr allgemeinen Annahmen (die Zweipunktfunktion muss nicht einmal positiv oder temperiert sein). Darüber hinaus liegt Mikrokausalität den Dispersionsbeziehungen für die S-Matrix und dem daraus resultierenden Hochenergieverhalten zugrunde, was in Collider-Experimenten (Froissart-Grenzen etc.) überprüft werden kann.
In gekrümmten Raumzeiten sind Argumente, die auf formalen Funktionsintegralen und der daraus resultierenden formalen Störungstheorie basieren, noch verdächtiger, da es im Allgemeinen weder eine Wick-Rotation noch eine S-Matrix gibt - die Geometrie der Raumzeit schließt dies seither aus Möglicherweise gibt es überhaupt keine globalen Isometrien. In diesem Fall ist ein algebraischer Ansatz eher geeignet.
Danke für deine Kommentare. Ich habe eine letzte Frage. Wenn wir wissen, dass für eine Theorie die S-Matrix Lorentz-invariant ist, folgt dann automatisch Mikrokausalität?
@NirmalyaKajuri: es scheint, ja, das tut es. Aus der Lorentz-Invarianz von S -Operator folgt, dass Interaktionshamiltonianer in raumartigen Intervallen pendeln, was nichts anderes als ein Mikrokausalitätskriterium ist.
@NameYYY Streng genommen existieren Interaktions-Hamiltonianer in QFT mit unendlichem Volumen nach Haags Theorem nicht (siehe z. B. physical.stackexchange.com/questions/87857/… ), daher ist dieses Argument nicht streng. Ein richtiges Argument müsste mithilfe von LSZ-Reduktionsformeln oder ähnlichem aufgestellt werden - und auch hier ist mir ein solches Argument nicht bekannt (ich würde es natürlich gerne wissen).
@PedroLauridsenRibeiro Parastatistische Feldtheorien respektieren die relativistische Kausalität, sind jedoch frei von Mikrokausalitätseinschränkungen.
@NanashiNoGombe wahr, parastatistische Feldmodelle verpflichten sich nicht zur Bose / Fermi-Mikrokausalität (nur beobachtbare zusammengesetzte Felder darin), können aber Lorentz-kovariant sein. Danke (wenn auch etwas spät!)

Betrachten Sie eine Poincare-invariante Skalarfeldtheorie. Angenommen, wir haben eine garantierte Poincare-Invarianz in dem Sinne, dass es eine einheitliche Darstellung der Poincare-Gruppe gibt, die auf den Hilbert-Raum wirkt, der den Skalarfeldoperator transformiert ϕ ( X ) wie

U ( G ) ϕ ( X ) U ( G ) = ϕ ( G X )
für Elemente G der Poincare-Gruppe.

Betrachten Sie zwei raumartig getrennte Punkte X , j . Wir wollen zeigen

[ ϕ ( X ) , ϕ ( j ) ] = ? 0.
Andererseits wissen wir das für zwei beliebige Punkte X ' = ( 0 , X ' ) Und j ' = ( 0 , j ' ) zum Zeitpunkt T = 0 , wir haben
[ ϕ ( 0 , X ' ) , ϕ ( 0 , j ' ) ] = 0 ,
was unmittelbar aus der Wellenfunktions- oder Gitterperspektive folgt; zB in einer Gittertheorie die Operatoren ϕ ( 0 , X ' ) Und ϕ ( 0 , j ' ) auf unterschiedliche Gitterplätze einwirken und somit pendeln.

Hier ist der entscheidende Schritt: Beachten Sie das für alle spacelike X , j , gibt es eine Poincare-Transformation G nehmen X , j Zu X ' , j ' , dh in einen Rahmen transformieren, in dem sich beide Punkte befinden T = 0 . also dann

U ( G ) [ ϕ ( X ) , ϕ ( j ) ] U ( G 1 ) = [ ϕ ( 0 , X ' ) , ϕ ( 0 , j ' ) ] = 0
und somit [ ϕ ( X ) , ϕ ( j ) ] = 0 wie gewünscht.

(In einer anderen Antwort erwähnt Pedro Ribeiro Lorentz-invariante Theorien, die nicht mikrokausal sind, und ich bin mir nicht sicher, wie diese gegen meine Annahmen verstoßen.)